Author Topic: EM Drive Developments - related to space flight applications - Thread 3  (Read 3131414 times)

Offline OttO

  • Member
  • Posts: 82
  • France
  • Liked: 92
  • Likes Given: 11

...
A gradient force in the waveguide, the evanescent wave propagates at a frequency which is less than the cutoff frequency.

How did you find it?

A google search with "evanescent waves momentum cutoff"  8)

Offline JasonAW3

  • Senior Member
  • *****
  • Posts: 2443
  • Claremore, Ok.
  • Liked: 410
  • Likes Given: 14
Question;

     Has there been any experiments utilizing materials other than copper as the primary construction material of this device?  It would be interesting and likely informative to see how other material constructions affect the performance of the device.

Jason
My God!  It's full of universes!

Offline OttO

  • Member
  • Posts: 82
  • France
  • Liked: 92
  • Likes Given: 11
I had a thought, probably stupid but...

IF we think about our little friends the photons inside the cavity.
After being injected they are going from one mirror to the other (at least before losing their initial road due to the imperfections of surface and other things).

On their return path they meet other ones.

We know that tapered wave guide are used to make analog of optical black and white holes.
There has been several experiences of hawking radiation emitting in tapered wave guides.

We know that energy gradient in the cavity can change the curvature of space (and wave lengths ?)
We know that dielectric modify the speed of light (and wave lengths ?). perhaps in our context these two effects can be seen as equivalent (of different order).

Going back to the photon:
When he meet the moving black hole horizon wave what happens?
Does he lost his hairs?

Is there a recoil?







Offline Mulletron

  • Full Member
  • ****
  • Posts: 1150
  • Liked: 837
  • Likes Given: 1071
http://www.jetpletters.ac.ru/ps/1407/article_21374.shtml

Chiral anomaly and the law of conservation of momentum in 3He-A.

And I can feel the change in the wind right now - Rod Stewart

Offline rfmwguy

  • EmDrive Builder (retired)
  • Senior Member
  • *****
  • Posts: 2205
  • Liked: 2713
  • Likes Given: 1134
I think it is important for all to remember that the EM drive is a physical system, not a mathematical one. In the physical system, cutoff is not a line in the sand that you shall not cross, rather it is (probably) the center of a range where propagation drops below some relative value of db. The EM drive will do as it does over a range of frequencies, plus or minus, just some will do better than others.

Add: Its also important that the magnatron drive is a noisy source so the cavity will select its own operating frequency. It would be nice to have the maximum power transfer from source to cavity but very often "Perfect" is the enemy of "Good enough."

Classic definition of cutoff is 3db. A single cavity will not have a steep shape factor in stopbands, no brickwall is correct. Return loss will be much more transitional in the passband... IOW i'd design and tune for best S11 performance at center frequency simply to keep the signal source protected as a matter of safety and efficiency. Also the bessel function has a shallower shape factor. Its best known characteristic is flat group time delay in passband for radar/pulse applications. Really, the frustum is a poor bandpass, being so assymetrical around a center frequency....but maybe that's part of the mystery  ;)

These high Q frustums may be difficult to deal with.

As example assuming 3.85GHZ resonance at Q = 60,000. Bandwidth at -3db points of 64kHz or 32kHz either side of ideal resonance. Then assuming we wish to operate at max 50% of that deviation, we need to hold excitation frequency to +-16kHz of the ideal and at the same time track resonate changes due to thermal expansion.

This is doable but not so easy as blasting away with a wide band magnetron into a lower Q cavity with flat end plates as against spherical end plates and a narrow band Rf generator.

Will shortly present the system I'm putting together to enable resonance tracking as the frustum warms up and alters it's length and end plate diameters.

Intend to use a very slow sweep spectrum analyser, over a tight frequency range, via an Rf coax switch that samples what is happening to the cavity via the same antenna that excites the cavity, then adjusts Rf frequency to get close, then use real time closed loop thrust feedback to centre the external Rf to the centre of the thrust curve.

Also intent to do thrust bandwidths to see how much external freq variance affects generated thrust, at the same -3db down points. So will measure both conventional bandwidth and thrust bandwidth. Then will start to get a feel for how sensitive this beast is to frequency variance versus generated thrust.

Using a wide band magnetron Rf source, there is little chance of being able to directly measure thrust bandwidth or even cavity Rf Q. So while the narrow band pathway will be slower to get there, it should yield much more interesting data.

Well stated...my experience is with Qs far less "astronomical" than 60K, which is seldom seen in the real world. This does pose the problem of fractional 3dB BW percentages, this means the risk of thermal drift; implying the need for "constant tuning" of the frustum. Perhaps the better solution is to design for lower Q considering all the thermal coefficients in play...especially at higher power levels. Definitely a trade-off to consider.
« Last Edit: 06/01/2015 01:24 pm by rfmwguy »

Offline rfmwguy

  • EmDrive Builder (retired)
  • Senior Member
  • *****
  • Posts: 2205
  • Liked: 2713
  • Likes Given: 1134
...

What are you suggesting here, that EagleWorks didn't actually run the cavity at the resonance? From what I recall they had actually measured the Q with a sense antenna. There's a well known formula for calculating resonant modes of a truncated cone. It is certainly correct - tested to death in many practical devices. And they're also tuning the frequency to hit the actual resonance if the shape is a little off (due to thermal heating for instance).

The point missing is, if you tune it for the highest Q and resonance, it reduces the thrust. I've proven to myself anyway, that the thrust happens due to the interference between the standing wave k and the evanescent wave Beta, phase factors.  Where they interfere is where the phase shift is happening due to attenuation, as it propagates into the small end. Optimal thrust will occur when the amplitude of the standing waves is nearly the same as the amplitude of the evanescent waves and the two are out of phase. If you concentrate only on higher Q at resonance like EW did, it will minimize the evanescent waves that drive the thrust. Therefore, a lower Q is more likely to have positive results. Perhaps @TheTraveler's point is true, that had EW tested it at the Df frequency, it may have provided more thrust. So nothing is falsified by their test except the idea that they understand how it works.

I'm still trying to crunch all this into design equations that are hopefully, more accurate and informative. It may take me a while.

Todd

Again, this doesn't make any sense.   When the RF that is coupled into a cavity is at the resonant frequency the Q will be maximal.   That means the return loss is also at a maximum and almost all the power goes into the cavity.   Any frequency that results in a lower Q will have a lower return loss and so there will be more power reflected back from the cavity.  If the return loss is 10 dB lower at the frequency with a lower Q and "better for producing thrust"  then the effective RF power transmitted to the cavity is just 1/10 of the power transmitted to the cavity at the resonant frequency where the Q is maximal.   So maybe less RF power is the key.   Reduce the RF power and get more thrust.   Reduce it further and get even more thrust...  ad infinitum until with no power you get infinite thrust.

Yes! Which begs the question whether the effect is understood. A 10x reduction of energy logically results in a ~10x reduction of performance...of course, logic may have nothing to do with the effect ;)

Offline Rodal

  • Senior Member
  • *****
  • Posts: 5911
  • USA
  • Liked: 6124
  • Likes Given: 5564
...

What are you suggesting here, that EagleWorks didn't actually run the cavity at the resonance? From what I recall they had actually measured the Q with a sense antenna. There's a well known formula for calculating resonant modes of a truncated cone. It is certainly correct - tested to death in many practical devices. And they're also tuning the frequency to hit the actual resonance if the shape is a little off (due to thermal heating for instance).

The point missing is, if you tune it for the highest Q and resonance, it reduces the thrust. I've proven to myself anyway, that the thrust happens due to the interference between the standing wave k and the evanescent wave Beta, phase factors.  Where they interfere is where the phase shift is happening due to attenuation, as it propagates into the small end. Optimal thrust will occur when the amplitude of the standing waves is nearly the same as the amplitude of the evanescent waves and the two are out of phase. If you concentrate only on higher Q at resonance like EW did, it will minimize the evanescent waves that drive the thrust. Therefore, a lower Q is more likely to have positive results. Perhaps @TheTraveler's point is true, that had EW tested it at the Df frequency, it may have provided more thrust. So nothing is falsified by their test except the idea that they understand how it works.

I'm still trying to crunch all this into design equations that are hopefully, more accurate and informative. It may take me a while.

Todd

Again, this doesn't make any sense.   When the RF that is coupled into a cavity is at the resonant frequency the Q will be maximal.   That means the return loss is also at a maximum and almost all the power goes into the cavity.   Any frequency that results in a lower Q will have a lower return loss and so there will be more power reflected back from the cavity.  If the return loss is 10 dB lower at the frequency with a lower Q and "better for producing thrust"  then the effective RF power transmitted to the cavity is just 1/10 of the power transmitted to the cavity at the resonant frequency where the Q is maximal.   So maybe less RF power is the key.   Reduce the RF power and get more thrust.   Reduce it further and get even more thrust...  ad infinitum until with no power you get infinite thrust.

The Brady report shows these cases where a lower Q produced a higher force.  The force more than doubled with the Q at 40% of the high value:

page 18
Table 2. Tapered Cavity Testing: Summary of Results

Mode    Frequency(MHz) ,      Q     Input Power (W) Peak Thrust (μN)  Number of Test Runs
TM211 1932.6                  7,320  16.9                   116.0                  5
TM211 1936.7                18,100  16.7                     54.1                  2

Same mode, (practically) same frequency and power  :

Notice that at a Q only 2/5 of the higher one (7320 instead of 18100) resulted in greater than twice as high a thrust force (116 instead of 54).

This experiment is important, as it shows that it is unwarranted to assume that there is monotonic one-to-one functional relationship for measured

 thrust = function (Q, inputPower, frequency).

This experiment falsifies such a relationship (also notice that the above data is composed of several test runs)

Perhaps this explains why we haven't seen any levitation (and much less flying cars) yet from Shawyer's superconducting EM Drive.


NOTE: I think that mode shape is the equivalent of TM212 in a cylinder (there is no TM210 mode in a truncated cone).  There is no international convention on how to number the mode shapes of a truncated cone.
« Last Edit: 06/01/2015 02:31 pm by Rodal »

Offline rfmwguy

  • EmDrive Builder (retired)
  • Senior Member
  • *****
  • Posts: 2205
  • Liked: 2713
  • Likes Given: 1134
...

What are you suggesting here, that EagleWorks didn't actually run the cavity at the resonance? From what I recall they had actually measured the Q with a sense antenna. There's a well known formula for calculating resonant modes of a truncated cone. It is certainly correct - tested to death in many practical devices. And they're also tuning the frequency to hit the actual resonance if the shape is a little off (due to thermal heating for instance).

The point missing is, if you tune it for the highest Q and resonance, it reduces the thrust. I've proven to myself anyway, that the thrust happens due to the interference between the standing wave k and the evanescent wave Beta, phase factors.  Where they interfere is where the phase shift is happening due to attenuation, as it propagates into the small end. Optimal thrust will occur when the amplitude of the standing waves is nearly the same as the amplitude of the evanescent waves and the two are out of phase. If you concentrate only on higher Q at resonance like EW did, it will minimize the evanescent waves that drive the thrust. Therefore, a lower Q is more likely to have positive results. Perhaps @TheTraveler's point is true, that had EW tested it at the Df frequency, it may have provided more thrust. So nothing is falsified by their test except the idea that they understand how it works.

I'm still trying to crunch all this into design equations that are hopefully, more accurate and informative. It may take me a while.

Todd

Again, this doesn't make any sense.   When the RF that is coupled into a cavity is at the resonant frequency the Q will be maximal.   That means the return loss is also at a maximum and almost all the power goes into the cavity.   Any frequency that results in a lower Q will have a lower return loss and so there will be more power reflected back from the cavity.  If the return loss is 10 dB lower at the frequency with a lower Q and "better for producing thrust"  then the effective RF power transmitted to the cavity is just 1/10 of the power transmitted to the cavity at the resonant frequency where the Q is maximal.   So maybe less RF power is the key.   Reduce the RF power and get more thrust.   Reduce it further and get even more thrust...  ad infinitum until with no power you get infinite thrust.

The Brady report shows these cases where a lower Q produced a higher force.

Mode    Frequency(MHz) ,      Q     Input Power (W) Peak Thrust (μN)
TM211 1932.6                  7,320  16.9                   116.0
TM211 1936.7                18,100  16.7                     54.1

Same mode, (practically) same frequency and power  :

Notice that reducing the Q to only 2/5 of the higher one (7320 instead of 18100) resulted in greater than twice as high a thrust force (116 instead of 54)

(...)

Therefore...a Q reduction widens the passband, making return loss (S11) broader, thus negating thermal drift and the need for tuning either the CF or the cavity. Prediction, real world factors (experimentation) will help define the Q versus thrust effect. I am not on the team pushing massive, unrealistic Q, especially with relatively unstable magnetron output.

Offline TheTraveller

Interesting email corro with Roger Shawyer. There is much useful information here.

Quote
Hi Traveller

Your proposals sound fine to me.

Note that the Q you achieve will also be dependent on how well you tune and match the impedance of the input antenna. We have used probe, loop and waveguide iris plates as input circuits. All have their own problems, but you should first calculate the wave impedance of the cavity at the input position. Standard text book equations work, as they always do. You can then design your chosen input circuit to match the wave impedance at the cavity resonant frequency.

All successful EmDrive thrusters that I know of have incorporated a tuning element of some sort at the input. Also no successful design used COMSOL without correction, as the software does not seem to cope with conditions close to cut-off.

Best regards
Roger

> Hi Roger,
>
> Thanks again for the assistance.
>
> With my EM Drive Calculator now matching your numbers, I'm starting to get
> a good gut feel for how the variables interact. Thanks again for your advise.
>
> It is beyond me why EW never took the time to do this and instead used
> COMSOL to determine their cavity resonance frequencies.
>
> The copper pieces for the frustum will be laser cut from 1mm thick
> sheet, professionally rolled / formed for the spherical end caps and the 2
> end flanges turned on a lathe to match their inside diameter and slope to
> that of the frustum side walls. All joints will be silver soldered. Any
> silver solder that gets inside the butt side wall joint or flanges will be
> removed before all internal surfaces are highly polished. External end caps
> will also be produced to clamp the spherical end plates between the flanges
> and the end caps.
>
> I assume it is ok to apply a protective anti oxidation film over the outer
> frustum surfaces? Of course none inside the frustum, just highly polished
> copper.
>
> If after the Flight Thruster replicator frustum is built and a spectrum
> scan is run across your suggested 3.9003GHz resonance frequency +- 250
> MHz should I expect to see and be able to measure the cavity Q at the
> resonant frequency?
>
> I assume you measure frustum Q as the bandwidth at the -3bd points divided
> into the centre frequency?
>
« Last Edit: 06/01/2015 02:54 pm by TheTraveller »
It Is Time For The EmDrive To Come Out Of The Shadows

Offline rfmwguy

  • EmDrive Builder (retired)
  • Senior Member
  • *****
  • Posts: 2205
  • Liked: 2713
  • Likes Given: 1134
@Rodal

Doc, so I haven't lost my way, what is your recommendation for frustum diameters and height using a conventional 2.45 GHz magnetron and the TM mode of your choosing? Thanks...

Offline Rodal

  • Senior Member
  • *****
  • Posts: 5911
  • USA
  • Liked: 6124
  • Likes Given: 5564
Interesting email corro with Roger Shawyer. There is much useful information here.

Quote
...Also no successful design used COMSOL without correction, as the software does not seem to cope with conditions close to cut-off.

Best regards
Roger
...
>
Discordant dissonant comment about COMSOL Finite Element Analysis software "not seeming to cope with conditions close to cut-off" falsified, for example, by this peer-reviewed article (hat tip to Otto):

http://www.jpier.org/PIER/pier151/07.15022404.pdf

where the authors of the peer reviewed article calculate how a particle in a rectangular waveguide can be pulled towards the light source or pushed away from the light source just by varying the frequency around the waveguide cutoff frequency.   All of the fields and forces analytical calculations were validated using COMSOL Multiphysics Finite Element Analysis.  The peer-reviewed literature contains numerous other reference of analysts successfully using COMSOL to analyze conditions below, near and above cut-off.

COMSOL Finite Element Analysis is a very powerful tool with a huge number of modules available.  COMSOL, as well as ANSYS, ABAQUS, ADINA, and other multiphysics packages are routinely used for such analysis at top companies, universities and research institutions (like CERN).  Most large companies and research institutions have their own finite element packages as well.

Just like any tool, what an analyst may achieve depends on the expertise and experience of the analyst and the COMSOL modules available, as well as their ability to write user-written subroutines (instead of just using a software package as a black box) .
« Last Edit: 06/01/2015 03:32 pm by Rodal »

Offline Rodal

  • Senior Member
  • *****
  • Posts: 5911
  • USA
  • Liked: 6124
  • Likes Given: 5564
@Rodal

Doc, so I haven't lost my way, what is your recommendation for frustum diameters and height using a conventional 2.45 GHz magnetron and the TM mode of your choosing? Thanks...
Why would you choose a transverse magnetic mode shape? Are you focusing on using dielectric inserts?

If your aim is replication, since Prof. Yang has achieved the highest reported thrust forces, a good way to start would be by replicating her published experiments (she gives the lengths.  The diameters have to be obtained from interpolation in her graphs.  Alternatively you can try asking her for the dimensions).
She uses mainly transverse electric modes, instead of the transverse magnetic mode you are proposing.

I understand that this (focusing on Yang's experiments) is what Kurt Zeller (@zellerium) and Brian Kraft from Cal Poly are planning to do.
« Last Edit: 06/01/2015 03:34 pm by Rodal »

Offline TheTraveller

Interesting email corro with Roger Shawyer. There is much useful information here.

Quote
...Also no successful design used COMSOL without correction, as the software does not seem to cope with conditions close to cut-off.

Best regards
Roger
...
>
Discordant dissonant comment about COMSOL Finite Element Analysis software "not seeming to cope with conditions close to cut-off" falsified by this peer-reviewed article (hat tip to Otto):

http://www.jpier.org/PIER/pier151/07.15022404.pdf

Where the authors of the peer reviewed article calculate how a particle in a rectangular waveguide can be pulled towards the light source or pushed away from the light source just by varying the frequency around the waveguide cutoff frequency.   All of the fields and forces analytical calculations were validated using COMSOL Multiphysics Finite Element Analysis.  The peer-reviewed literature contains numerous other reference of analysts successfully using COMSOL to analyze conditions below, near and above cut-off.

COMSOL Finite Element Analysis is a very powerful tool with a huge number of modules available.  COMSOL, as well as ANSYS, ABAQUS, ADINA, and other multiphysics packages are routinely used for such analysis at top companies, universities and research institutions (like CERN).  Most large companies and research institutions have their own finite element packages as well.

Just like any tool, what an analyst may achieve depends on the expertise and experience of the analyst and the COMSOL modules available, as anything else.

Sounds like an advert for COMSOL. Do you sell or rep or support it?

Note well Roger Shawyers experience driven advise:

Quote
Also no successful design used COMSOL without correction, as the software does not seem to cope with conditions close to cut-off.

With my EM Drive Calculator, which has been checked against SPR's in house EM Drive design software, there seems little need for COMSOL.

As advised, I'm working with microwave industry equations to define the best side wall antenna location, antenna design and tuning system to excite TE103 mode and get impedance matching. Once that is checked out, I'll add it to the EM Drive Calculator as I hate others needing to reinvent the wheel.
« Last Edit: 06/01/2015 03:25 pm by TheTraveller »
It Is Time For The EmDrive To Come Out Of The Shadows

Offline Notsosureofit

  • Full Member
  • ****
  • Posts: 691
  • Liked: 747
  • Likes Given: 1729
http://www.jetpletters.ac.ru/ps/1407/article_21374.shtml

Chiral anomaly and the law of conservation of momentum in 3He-A.

Thank you Mulletron !

This is a particularly good article, not because of He3, but because he elaborates on the interaction of the Bose particles (the photon standing waves in our case) and the Fermi (principally the electrons in the cavity wall) to show the existence of a momentum current term in the interaction.  This does remind me of the current term in the Sachs-Schwebel quaternion formulation for GR. 

Offline Rodal

  • Senior Member
  • *****
  • Posts: 5911
  • USA
  • Liked: 6124
  • Likes Given: 5564
http://www.jetpletters.ac.ru/ps/1407/article_21374.shtml

Chiral anomaly and the law of conservation of momentum in 3He-A.

Thank you Mulletron !

This is a particularly good article, not because of He3, but because he elaborates on the interaction of the Bose particles (the photon standing waves in our case) and the Fermi (principally the electrons in the cavity wall) to show the existence of a momentum current term in the interaction.  This does remind me of the current term in the Sachs-Schwebel quaternion formulation for GR.

Notsosureofit , could you please explain as to whether this interaction, and the existence of this momentum current term could relate (somehow) to your Notsosureofit hypothesis for the EM Drive or the other possible explanations put forward ?

Concerning your example of the photon standing waves, would the interaction also take place with evanescent waves, as contemplated by WarpTech?

Thanks
« Last Edit: 06/01/2015 03:48 pm by Rodal »

Offline Rodal

  • Senior Member
  • *****
  • Posts: 5911
  • USA
  • Liked: 6124
  • Likes Given: 5564
...Sounds like an advert for COMSOL. Do you sell or rep or support it?...
No, I don't sell or support COMSOL. I have expertise in Finite Element Analysis.

Does Shawyer have expertise in Finite Element analysis to make the negative comment about COMSOL FEA that you posted in this thread http://forum.nasaspaceflight.com/index.php?topic=37642.msg1382951#msg1382951 ?

Concerning you comment about "sounding like and ad" this is quite a statement coming from somebody that periodically inundates this thread with praise about Shawyer and his company.

Wish that you would get going with your experiment and not post any further negative comments from Shawyer about other companies (COMSOL for example), which adds up to Shawyer's negative comment to the media about NASA and about the West.
« Last Edit: 06/01/2015 03:59 pm by Rodal »

Offline Notsosureofit

  • Full Member
  • ****
  • Posts: 691
  • Liked: 747
  • Likes Given: 1729
http://www.jetpletters.ac.ru/ps/1407/article_21374.shtml

Chiral anomaly and the law of conservation of momentum in 3He-A.

Thank you Mulletron !

This is a particularly good article, not because of He3, but because he elaborates on the interaction of the Bose particles (the photon standing waves in our case) and the Fermi (principally the electrons in the cavity wall) to show the existence of a momentum current term in the interaction.  This does remind me of the current term in the Sachs-Schwebel quaternion formulation for GR.

Notsosureofit , could you please explain as to whether this interaction, and the existence of this momentum current term could relate (somehow) to your Notsosureofit hypothesis for the EM Drive or the other possible explanations put forward ?

Concerning your example of the photon standing waves, would the interaction also take place with evanescent waves, as contemplated by WarpTech?

Thanks

Only a quick look but eq 12 and following coupling constant is the interesting point.  This is different then the photon-photon Gr interaction we were looking at previously.  I'll try to get the brain thinking after work.

Added: A comment by Schwinger: "Incidentally, the probability of actual pair creation is obtained from the imaginary part of the electromagnetic field action integral. " ... for S. White ???
« Last Edit: 06/01/2015 04:40 pm by Notsosureofit »

Offline Rodal

  • Senior Member
  • *****
  • Posts: 5911
  • USA
  • Liked: 6124
  • Likes Given: 5564
..
Added: A comment by Schwinger: "Incidentally, the probability of actual pair creation is obtained from the imaginary part of the electromagnetic field action integral. " ... for S. White ???
I took a peek at the paper and I noticed at the very beginning stating "The nonconservation of momentum of the fermion vacuum in superfluid 3He-A at T = 0 is the result of chiral anomaly which is completely analogous to the axial current anomaly in quantum chromodynamics" [The vacuum in QCD is a linear combination of an infinite number of vacua with different winding numbers.  http://arxiv.org/pdf/0809.0212v2.pdf ]
 
So, is this an anomaly occurring only near absolute zero with superfluids?

Liguid Helium when it turns supefluid has very odd properties, doing things that look impossible because it has zero viscosity

« Last Edit: 06/01/2015 06:29 pm by Rodal »

Offline aero

  • Senior Member
  • *****
  • Posts: 3629
  • 92129
  • Liked: 1146
  • Likes Given: 360
A question about Dr. White's QV conjecture. Just a partially formed thought.

If his analogy of the electrons-positrons at a square dance with dancers changing partners by popping into and out of the QV, then wouldn't this same idea result in a particle electron idea of quantum tunneling? That is, electrons approach a barrier and dive into the QV. Another electron pops out of the QV nearby, but it has a probability of being on the wrong side of the barrier sence the barrier does not penetrate the QV.

This mechanism might be an excuse for the near instantaneous transit of the barrier, currently measured to be on the scale of attoseconds (10-18 seconds). In that short interval of time even light speed only moves a tenth of a nanometer and barriers are much thicker than that.

Of course I know that the wave theory of tunneling has the electron wave already partially on the other side of the barrier as it reaches the barrier so the wave tunnels through to fully re-form with the part that was already on the other side. That is reasonable for photons, but electrons have rest mass and that causes confusion to me.
« Last Edit: 06/01/2015 05:56 pm by aero »
Retired, working interesting problems

Offline zellerium

  • Full Member
  • *
  • Posts: 172
  • Pittsburgh, PA
  • Liked: 283
  • Likes Given: 402
Hey everyone,

Here is a summary of our current experimental design.
Any feedback is welcome and appreciated.  :)

Kurt

Tags:
 

Advertisement NovaTech
Advertisement Northrop Grumman
Advertisement
Advertisement Margaritaville Beach Resort South Padre Island
Advertisement Brady Kenniston
Advertisement NextSpaceflight
Advertisement Nathan Barker Photography
0